LSAT and Law School Admissions Forum

Get expert LSAT preparation and law school admissions advice from PowerScore Test Preparation.

 Administrator
PowerScore Staff
  • PowerScore Staff
  • Posts: 8916
  • Joined: Feb 02, 2011
|
#23995
Complete Question Explanation

Weaken. The correct answer choice is (E)

The prediction presented here is a simple one: that if oil prices fall by half, then gasoline prices will fall by half. The question stem which follows requires us to find the answer choice which calls this prediction into question. The correct answer will give reason to doubt the associated fall in gas prices.

Answer choice (A): A reduced use of gasoline by “some” consumers would not help to refute the prediction that falling oil prices would be accompanied by proportionate falling gas prices, so this answer choice is incorrect.

Answer choice (B): This choice would actually strengthen the argument in the stimulus, ruling out one factor that might throw off the proportionality between oil prices and gas prices. That is, if we can count on the gas makers to avoid increasing their profit margins, then it seems more likely that a drop in oil prices would be accompanies by a proportionate drop in gas prices.

Answer choice (C): The presence of a competitive gas market would not refute the prediction made in the stimulus. In fact, this choice strengthens the argument: if many gas companies are competing, then this increases the likelihood that a drop in oil prices would translate into a similar drop in gas prices (rather than an increase in gas makers’ profits, for example).

Answer choice (D): The information presented in this choice is extremely vague, and fails to refute the prediction presented in the stimulus. Basically, this information give us no reason to question the argument that a 50% drop in oil prices will be accompanied by a 50% drop in gas prices.

Answer choice (E): This is the correct answer choice. Here we are provided with a list of fixed costs which come into play regardless of oil prices. The presence of such costs means that gas prices consider much more than just the price of oil. This makes it unlikely that the drop in oil prices will be mirrored by a drop in gas prices.
 scharles35
  • Posts: 15
  • Joined: Apr 19, 2014
|
#14851
The stimulus reads:
Oil analyst predicts that if the price of oil falls by half, the customer's purchase price of gasoline made from this oil will also fall by half.

In the back of the book it said this this question was a Must Be True question. How is this question which states "Which one of the following, if true, would cast the most serious doubt on the prediction made by the oil analysts? a must be true question i thought it was a weaken question type. I had answered (D). Can someone please help explain how this is a Must Be True question and how the correct answer is (E)
 Lucas Moreau
PowerScore Staff
  • PowerScore Staff
  • Posts: 216
  • Joined: Dec 13, 2012
|
#14862
Hello, Charles,

That's a good point - that question really does look like a Weaken question! It's possible that the question was identified wrongly in the book. :0 Thanks for pointing that out, I'll email people about it right away.

As far as the question itself, answer choice D is talking about the amount of gasoline purchased, which doesn't come up at all in the stimulus. The stimulus only talks about a link between the price of oil and the consumer's purchase price of gasoline - how much the consumer purchases doesn't enter into it! :)

Answer choice E, if true, states that the price of gasoline won't fall exactly as much as the price of oil, since the price of gasoline is dependent on many other things than just the price of oil.

Here's an example: let's say oil costs $10 a barrel and is $1 a gallon at the pump. (Heh, I wish.) And let's say that that price at the pump is $0.40 made up of the price of the actual oil and $0.60 made up of the extra costs spoken of in answer choice E.

If the price of oil then falls to $5 a barrel, then the price at the pump will only fall to $0.80 a gallon at the pump as the oil price portion shrinks to $0.20, because the $0.60 isn't changed by the decrease in the oil price. Does that make sense? :)

Hope that helps,
Lucas Moreau
 scharles35
  • Posts: 15
  • Joined: Apr 19, 2014
|
#14867
Thanks so i wasn't going crazy!!it was a weaken question lol and once again thank you for you help!
 MannyH
  • Posts: 10
  • Joined: Jun 19, 2018
|
#47061
In the LRB it states that "To weaken a conditional conclusion, attack the necessary condition by showing that the necessary condition does not need to occur in order for the sufficient condition to occur". To my interpretation the necessary condition in the stimulus is "..the purchase price for gasoline made from the oil will be half". This led me to choose choice B over my other contender answer choice E which proved to be the correct answer. Can you please provide a little more clarity on how to utilize the strategy of using the necessary condition to weaken a conditional conclusion?
 James Finch
PowerScore Staff
  • PowerScore Staff
  • Posts: 943
  • Joined: Sep 06, 2017
|
#47106
Hi Manny,

Just to clarify a bit, weakening a conditional relationship requires first identifying the conditional, then showing that when the sufficient condition occurs, the necessary condition doesn't. Here we are given:

Oil Price Halved (OPH) :arrow: Gas Price Halved (GPH)

So we need to show

OPH :arrow: GPH

Answer choice (B) actually supports the conclusion, by saying that the profit margin will be the same. As oil is the main constituent of gasoline and its cost goes down, if the profit margin remains the same then gas prices must fall as well (not necessarily by half, but they'll still definitely fall).

Answer choice (E) works by making the price of gas somewhat independent from that of oil by adding in other constituent elements that remain constant regardless of oil price. This means that halving only one of the factors in price, oil, will not lead to a halving of the gas price because the other factors would remain the same, thus:

OPH :arrow: GPH

Hope this helps!
 deck1134
  • Posts: 160
  • Joined: Jun 11, 2018
|
#48005
Hi all,

I chose answer choice D on this question. In previous LSAT questions we have had to have a working knowledge of supply and demand. If demand increases, price will stay artificially inflated beyond input forces (i.e. price of oil falls). This knowledge seems to be in line with what Law Services expects us to know. If that is the case, then we could reasonably infer that if demand increases, (amont purchased rises) then the purchase price will not fall by half. It may fall by 49.9%, but not half. Isn't that a weaken answer?

I also thought that answer choice E is a poor answer choice. while there may be other products involved in the making of gas, it does not guarantee that there will be any corresponding change in price. How do we reach the conclusion?

Thanks!
 Adam Tyson
PowerScore Staff
  • PowerScore Staff
  • Posts: 5153
  • Joined: Apr 14, 2011
|
#48883
Hey there Deck! I think you may be reading a little too much into that stimulus. The concept of supply and demand aren't raised there, only the concepts of cost and price. Also, demand doesn't necessarily mean that the amount purchased goes up, but only that the amount that people desire to purchase goes up, or that there is upward pressure on the supply. Sometimes demand goes up because supply actually goes down!

Putting aside any sort of economics lesson, which I am completely unqualified to lead, let's deal with what the stimulus actually says. The author wants us to believe that if the price for oil drops, then the price of gasoline will drop by the same percentage. The price of oil is one cost factor in the price of gasoline, and the author seems to have assumed that it is the only cost factor and that is the only thing that affects the price of the final product. To weaken that argument, we might point out that there could be other factors that go into the price of gasoline. No need to bring up supply and demand for that analysis!

If a question directly raises an issue of supply and demand, you're correct that the authors of this test expect you to understand some of the very basic concepts implicated. If, however, they don't raise the issues of supply or demand or both, then it's probably not a good idea to bring in that outside knowledge and try to apply it to the things they did talk about, like cost, revenue, price, or profit. That's good general advice for most of the questions on this test! Try to deal with the information you are given as it is and without any outside information other than some very basic things, like if the sun is shining it's probably daytime. Stick to the text and you almost always will be on the right path.

Keep at it!
 dandelionsroar
  • Posts: 27
  • Joined: Oct 18, 2018
|
#60305
In answer choice E doesn't the phrase "none of which varies significantly with oil prices" kind of support the stimulus? Even though they have a factor in the pricing, if the usually are constant costs why would they greatly effect the relationship between the price of oil and the purchase price?

Thank you!!
 Zach Marino
PowerScore Staff
  • PowerScore Staff
  • Posts: 6
  • Joined: Oct 04, 2018
|
#60393
dandelionsroar wrote:In answer choice E doesn't the phrase "none of which varies significantly with oil prices" kind of support the stimulus? Even though they have a factor in the pricing, if the usually are constant costs why would they greatly effect the relationship between the price of oil and the purchase price?

Thank you!!
Hi! So, according to answer choice E, these factors don't vary significantly with oil prices as you stated. However, this means these factors, since they don't vary, make up a large portion of the consumer's price that will not be affected by oil prices. Therefore, to claim that consumer prices would fall proportionally by half as oil prices fall by half is not a logically sound conclusion to make.

Hope this helps!

Get the most out of your LSAT Prep Plus subscription.

Analyze and track your performance with our Testing and Analytics Package.